1 + 3 + 4 = 4 + 3 + 1. What property is being described here?

Associative Property of Addition

Commutative Property of Addition

Associative Property of Multiplication

Answers

Answer 1

Answer: Commutative property of addition

Step-by-step explanation:

Answer 2

[tex]\huge\textsf{Hey there!}[/tex]

[tex]\large\text{1 + 3 + 4 = 4 + 3 + 1}[/tex]

[tex]\large\text{1 + 3 + 4}[/tex]

[tex]\large\text{4 + 4}[/tex]

[tex]\large\text{= \bf 8}[/tex]

[tex]\large\text{4 + 3 + 1}[/tex]

[tex]\large\text{7 + 1}[/tex]

[tex]\large\text{= \bf 8}[/tex]

[tex]\large\text{\bf 8 = 8}[/tex]

[tex]\large\textsf{We could remove Option C. because there’s no sign of multiplication.}[/tex]

[tex]\large\textsf{This leaves us to Option A and Option B because the equation(s) has}\\\large\textsf{sum/addition}[/tex]

[tex]\large\textsf{Associative Property of Addition formula is:}[/tex]

[tex]\bullet\large\text{ \bf (a + b) + c = a + (b + c)}[/tex]

[tex]\large\textsf{Commutative Property of Addition formula is:}[/tex]

[tex]\bullet\large\text{ \ \bf a + b = b + a}[/tex]

[tex]\boxed{\boxed{\large\textsf{Answer: \huge \bf Commutative Property of Addition}}}\\\boxed{\huge\textsf{(Option B.)}}\huge\checkmark[/tex]

[tex]\large\textsf{Good luck on your assignment and enjoy your day!}[/tex]

~[tex]\frak{Amphitrite1040:)}[/tex]


Related Questions

How would the domain and range of the function y = one-fourth x minus 6 be determined? Explain.

Answers

Answer:

Domain = ( -∞ , ∞ )

Range = ( ∞ , ∞ )

Step-by-step explanation:

A function is given to us and we need to find the domain and range of the given function .

The function :-

[tex]\rm \implies y = \dfrac{1}{4}x - 6 [/tex]

Definitions :-

Range :- The range is the set of all valid y values .Domain :- All real numbers except where the expression is undefined.

In this case, there is no real number that makes the expression undefined. Therefore the domain will be :-

Domain :-

[tex]\rm Domain = ( -\infty , \infty ) [/tex]

or

[tex]\rm Domain = \{ x | x \in \mathbb{R} \}[/tex]

Range :-

[tex]\rm Range = ( -\infty , \infty ) [/tex]

or

[tex]\rm Range = \{ y | y \in \mathbb{R}\} [/tex]

Answer:

Create a table or a graph of the function. The domain represents all input values and the range represents all output values. The domain and range contain all real numbers.

Step-by-step explanation:

Type the correct answer in each box. Use numerals instead of words

Answers

Answer:

a) 1.73

b)1.32

Step-by-step explanation:

y = f(x) = 3^x

f(1/2) = 3^1/2 = √(3) = 1.73

f(1/4) = 3^1/4 = 1/4√3 = 1.32

Question 1: Use the image and your knowledge of the isosceles triangle to find the value of x

Answers

Answer:

35 degrees

Step-by-step explanation:

since both sides are equal, so are the angles involved.

Someone tell me where everyone is going right please !!

Answers

Answer:

A 2

B 1

C 4

D 3

Step-by-step explanation:

( ) means not including (also used for infinity)

[ ] means including

The answer choices are in interval form, showing all the possible answer choices between two numbers and if the answer choice includes or excludes a number using the () []

Explanation for A:

x < 7.8 means x won't include 7.8 since the symbol is "less than" and not "less than or equal to".

x can be anything, it just has to be less than 7.8, so the answer is:

2 (-infinity, 7.8)

Explanation for B:

x <= 7.8 means x will include 7.8 since the symbol means "less than or equal to".

x can be anything, it just has to be less than or equal to 7.8, so the answer is:

1 (-infinity, 7.8]

Explanation for C:

x >= 7.8 means x will include 7.8 since the symbol means "greater than or equal to".

x can be anything, it just has to be greater than or equal to 7.8, so the answer is:

4 [7.8, infinity)

Explanation for D:

x > 7.8 means x won't include 7.8 since the symbol means "greater than" and not "greater than or equal to".

x can be anything, it just has to be greater than 7.8, so the answer is:

3 (7.8, infinity)

Hope it helps (●'◡'●)

Answer:

A:2 B:1 C:4 D:3

Step-by-step explanation:

What is the phenomenon that many types of data analysis become significantly harder as the dimensionality of the data increases

Answers

Answer:

the curse of dimensionality

Step-by-step explanation:

the cause of dimensionality has to do with the different phenomena that comes up whenever data is to be analyzed or organized in high dimensional spaces which do not happen in low dimensional spaces. This curse can be explained simply as having error increase just as there are increases in the number of features. when the features we have is more than the observations, it could cause the model to be overfitted.

Tìm x €N: a) 24:(2x-4)+14=26

Answers

346788 ese es la ª
B a 135899933


Help and explain !!!!

Answers

i believe it's y-intercept!

to be honest i was never taught why it was the y-intercept, i was just told that it was. hope this helps!

if f(x) = 2x² +1 and g(x) = x² - 7 find (f+g) (x)

Help me find (f+g)(x) please!

Answers

Answer:

[tex](f+g)(x) = 3x^2 - 6[/tex]

Step-by-step explanation:

[tex]f(x) = 2x^2 + 1, \ g(x) = x^2 - 7\\\\(f +g)(x) = f(x) + g(x) \\\\(f+g)(x) = 2x^2 + 1 + x^2 - 7 \\\\(f+ g)(x) = 3x^2 - 6[/tex]

What percent did the price drop

Answers

Answer:

10%

Step-by-step explanation:

You have the correct answer. Nice job!

What percentage of 1hour is 6munites 20seconds

Answers

Answer:

10

Step-by-step explanation:

10 percentage for this question answer

what shape is this cause I'm having a bit of trouble?​

Answers

Answer:

isn't that a square or is it called sum eles-

Step-by-step explanation:

I think that’s a square because it has 4 equal sides

what are the x-coordinates of the solutions? Choose all that apply.

a. -2
b. 1
c. -1
d. 0

Answers

Answer:

a. -2

c. -1

Step-by-step explanation:

these are the only values for x, where the 2 equations deliver the same y value.

in case of a. y = -2×-2 - 1 = 4 - 1 = 3

and y = 3¹ = 3

c. y = -2×-1 - 1 = 2 - 1 = 1

y = 3⁰ = 1

List a function that's it's own inverse

Answers

Answer: f(x) = 3/x

==========================================================

Explanation:

If g(x) is the inverse of f(x), and vice versa, then we have these two properties:

f(g(x)) = xg(f(x)) = x

Since we want to find a function that is its own inverse, we want f(x) and g(x) to be the same function.

Through trial and error, you should find that f(x) = g(x) = x fit the description.

f(x) = x

f( g(x) ) = g(x) ... replace every x with g(x)

f( g(x) ) = x

You should find that g(f(x)) = x as well.

One possible answer is f(x) = x

---------------------------

Through more trial and error, you should find that f(x) = g(x) = 1/x works as well. In fact, anything of the form f(x) = g(x) = k/x will work.

The proof can be written as follows

f(x) = k/x

f( g(x) ) = k/( g(x) )

f( g(x) ) = k/( k/x )

f( g(x) ) = (k/1) divide (k/x)

f( g(x) ) = (k/1) * (x/k)

f( g(x) ) = x

Through similar steps, you should find that g(f(x)) = x is the case also.

This proves that f(x) = k/x is its own inverse, where k is a real number constant.

Another possible answer is anything of the form f(x) = k/x

If we pick k = 3, then we get f(x) = 3/x which is the answer I wrote above.

You can pick any k value you want.

---------------------------

There may be other types of functions that have this property, but I'm blanking on what they might be.

Please help me with this one I seriously suck at math

Answers

Answer:

6×5+8×6+6×7+2×1/2×4×8

= 152 in^2

○●○●○●○●○●○

☆Hope it helps...

question on the image
(with the steps)​

Answers

Answer:

40°

Step-by-step explanation:

The given triangle is a isosceles triangle . And we know that in a isosceles triangle opposite angles are equal . Therefore ,

> x + 70° + 70° = 180°

> x + 140° = 180°

> x = 180° - 140°

> x = 40°

Answer:

40 degrees

Step-by-step explanation:

We know that the base angles are congruent (a fancy word for equal) because of the slash marks. This means that both of the base angles are 70.

Every triangle adds up to 180 degrees.

So, You can just do 180 - (70+70)  

Algebra version:

70 + 70 + x = 180

140 + x = 180  (subtract 140 on both sides)

x = 40

hope this helps :)

What is the equation of the line graphed below?

Answers

Answer:

y=2/5x

Step-by-step explanation:

Rise over run: you rise by 2, run by 5. Or you can use the slope formula: m=y2-y1/x2-x1

2-0/5-0=2/5=m

find the value of the given equation (-155)+(-20) + (-14) + (-133) + (190) + (170) +(-73)​

Answers

Answer:

-37

Step-by-step explanation:

-155-20-14-133-73=-395

190+170=360

-395+360=-37

The sum of two numbers is 3000. If 8% of one number is equal to 12% of the other, find the numbers.​

Answers

one is 1800 and another is 1200

Step-by-step explanation:

let no's be A and B

so, A+B =3000

given, 8% = 8/100 = 2/25

12%= 12/100 = 3/25

As per question 8% of A is = 12% of B

so, (2/25)*A = (3/25)*B

now, multiply cross wise

then we get, (A/B) = 3:2

thus two numbers are in the ratio of 3:2

so, let A =3x, B=2x

cause sum of these is 3000 so, A+B =3000

3x+2x =3000

5x = 3000

x= 3000/5

.: x = 600

so, first no is 3x =3*600=1800

2nd no is 2x= 2*600= 1200

Hope this helps you

Find the length of side xx in simplest radical form with a rational denominator.

Answers

Answer:

Solution given:

Relationship between perpendicular and hypotenuse is given by sin angle

Sin 30°=opposite/hypotenuse

½=[tex]\frac{x}{9}[/tex]

x=[tex]\frac{9}{2}[/tex]

The value of x is [tex]\frac{9}{2}[/tex]

A straight line is drawn through the points A(1,1) and B(5,-2). Calculate the gradient

Answers

Answer:

-3/4

Step-by-step explanation:

The line passes through the two points which are A(1,1) and B(5,-2) . We know that the slope of the line passing through two points is ,

[tex]\implies Slope =\dfrac{y_2-y_1}{x_2-x_1}\\\\\implies Slope =\dfrac{ -2-1}{5-1}\\\\\implies Slope =\dfrac{-3}{4} \\\\\implies \underline{\underline{ Slope (m) =\dfrac{-3}{4}}}[/tex]

Hence the slope of the line is -3/4 .

The tables below show the values of y corresponding to different values of x: Table A x 3 3 2 y 1 0 0 Table B x 3 5 5 y −2 2 −2 Which statement is true for the tables? (1 point) Both Table A and Table B represent functions. Both Table A and Table B do not represent functions. Table A does not represent a function, but Table B represents a function. Table A represents a function, but Table B does not represent a function.

Answers

Answer:

is it possable you can show the graph

Step-by-step explanation:

Answer:

2nd option ,that is, Both tables do not represent a function.

Step-by-step explanation:

For a table to represent a function it's input value must correspond to exactly one output value.

As in Table A the input value of 3 corresponds to 2 different outputs, where

as in table B the input value of 5 corresponds to 2 different outputs.

x is taken as the input and y is taken as output for both the tables.

Therefore, Both tables dobnot represent a function.

Jamie's chemistry textbook weighs 7/10 of a pound and his geometry textbook weighs 3/5 of a pound. How much more does the chemistry textbook weigh than the geometry textbook?

Answers

Jaime’s chemistry textbook weighs 0.1 pound more than his geometry textbook. In fraction form, it is 1/10 pound more.

Choose the correct answer from the given four options:In an AP if a = –7.2, d = 3.6, an = 7.2, then n

2

4

3

5​

Answers

Answer:

n=5

Step-by-step explanation:

by using

n=(an-a)/a+1

substituting values

n= 7.2-(-7.2)/3.6 +1

n=5

Will mark brainlest helpppp pleaseeeee​

Answers

Answer:

come to mya massanger dk khanal fb user

Step-by-step explanation:

i will teach you by sending picture

Let ABCD and EFGH be two quadrilaterals such that ABCD ~ EFGH. If AB = 15 cm, EF = 18 cm and the perimeter of ABCD is 40 cm, find the perimeter of EFGH.​

Answers

Step-by-step explanation:

this photo is correct answer of this question.

Ling must spend no more than $40.00 on decorations for a party. She has spent $10.00 on streamers and wants to buy bags of balloons as well. Each bag of balloons costs $2.50. The inequality below represents x, the number of bags she can buy given the spending limit and how much she has already spent on streamers.

10 + 2.5 x less-than-or-equal-to 40

Which best describes the number of bags of balloons she can buy?

Answers

Answer:

She can buy from 0 to 20 bags, but no more.

Answer:

b <3

Step-by-step explanation:

Help and explain pls and ty

Answers

[tex]\displaystyle\bf (3-5\sqrt{5} )-(5\sqrt{3} -3)=3+3-5\sqrt{5} -5\sqrt{3} =\boxed{6-5\sqrt{5} -5\sqrt{3} } \\\\\\\\\sqrt{5} (\sqrt{5} -2x)=\boxed{-2x\sqrt{5} +5}[/tex]

Lily bought a pair of gloves and a shirt.
The gloves cost £4
She sold the gloves and the shirt for a total of £48
She made 100% profit on the cost of the gloves.
20% profit on the total cost.
Work out her percentage profit on the cost of the skirt.
Answer 1 decimal place.

Answers

Answer:

Profit % = 11,11 %

Step-by-step explanation:

Answer:

14% profit on shirt/skirt?...

actually as written the question can not be answered....

the start of the problem has gloves, and SHIRT... at the end you want the

profit on a SKIRT

Step-by-step explanation:

gloves sold for 8

shirt sold for 40

.2(48) = 9.60 profit

9.60 - 4 = 5.60 profit of shirt

x(40) = 5.60

14% profit on shirt

Find the measure of the indicated angle

Answers

Answer:

76°

Step-by-step explanation:

180-(2*52)

= 180-104

= 76

Answered by GAUTHMATH

what is the volume of the square pyramid? answer i need this done for my geometry credit thank you

Answers

Answer: 171.5

Step-by-step explanation:

1/2 x 7 x 7 x7  = 171.5 cm3

Answer:

[tex]volume=1/3(area ~of ~base) \times( height)[/tex]

[tex]v=1/3(7\times7)\times7[/tex]

[tex]v=343/3[/tex]

[tex]v=114[/tex]

-----------------------

hope it helps...

have a great day!!

Other Questions
HELP ASAP 10 POINTS AND BRAINLIEST Which number line model represents the expression 3.5+(-5) Fill in the blank with the correct choice:Neither the CEO nor the trustees _____ the outcome of the vote.A. knowB. to knowC. have knownD. are knowing What is the equation of the following line? All narratives ______, but they may use different elements to do so. PLZZ HELPPPPPP I NEED TO FINISH THIS IN 8 MINS find the compound interest on rs 800 for 2 years at 8% per annum interest being payable annually Conjugacin de los verbos irregulares al presente.1.Le club ___ la tte du classement. (PERDRE2. combien ___ tu ta maison ? (VENDRE)3.Ils nous ___ pour des imbciles ! (PRENDRE)4.Tu m___ la porte sil-te-plat ? (OUVRIR)5. Nous ___ des champignons dans les bois. (CUEILLIR)6.Nous te ___ toujours la mme chose (DIRE7. Tu ___ le journal Lquipe. (LIRE)8.Nous te ___ toujours la mme chose (DIRE) [tex]\frac{5}{8}[/tex] [tex]\frac{2}{3}[/tex] which is th main cereal crop of hill region? in nepal Katrina has long believed Eric Church is the best musician of all time. She just read a review that praised Eric Church for his musical genius. She is very confident in her perception, despite all the earlier reviews she had read that pointed out his limitations. Katrina is demonstrating the need:______.A) to reach closure quickly.B) to confirm preexisting beliefs.C) for accuracy.D) for validation. how would hemoglobin content differ in a person living in philadelphia compared to someone living in denver Is the function g(x)= 5x+4 linear or nonlinear A football quarterback runs 15.0 m straight down the playing field in 3.00 s. He is then hit and pushed 3.00 m straight backward in 1.71 s. He breaks the tackle and runs straight forward another 24.0 m in 5.20 s. Calculate his average velocity (in m/s) for the entire motion. (Assume the quarterback's initial direction is positive. Indicate the direction with the sign of your answer.) solve for x in the equation x^2-12+36=80 I forgot to label the triangle below! I just know that the cos A = 0.48. Based on this information, which angle should be marked A? The probability distribution for arandom variable x is given in the table.-5-3-2023Probability.17.113.133.16.11.10Find the probability that -2 The Brady & Matthew Camera Company has just come out with their newest professional quality digital camera, the ToughPix1. The company is selling this camera only through its new mobile app at a profit of $221 per camera. This purchase comes with a guarantee that, barring gross negligence, if the camera breaks in the first two years after purchase, Brady & Matthew will replace it free of charge. Replacing a camera in this way costs the company $4900. Suppose for each ToughPix1 there is a 3% chance that it will need to be replaced exactly once, a 2% chance that it will need to be replaced exactly twice, and a 95% chance that it will not need to be replaced. How do you think the presence of so many independent city-states in such close proximity could have contributed to the Renaissance beginning in Italy? I need to fill in the blanks with the correct Spanish word